REA - 161

Pataasin ang iyong marka sa homework at exams ngayon gamit ang Quizwiz!

Which of the following areas is not a type of assessment accommodation? (A) Presentation (B) Scheduling (C) Content (D) Timing

(C) Content Content will never be a type of accommodation because accommodations never alter content.

A teacher presents students with the following number pattern: 1, 3, 9, 27, 15. Given this pattern, the next number would be (A) 81. (B) 45. (C) 54. (D) 1.

(A) 81. The key to this number pattern is that each number is multiplied by 3 to calculate the next number. The final given number is 27, and since 27 multiplied by 3 equals 81

Which of the following curriculum development projects would a Special Education teacher be most likely to work with an SLP to develop? (A) A rubric for an oral presentation project (B) Calculator practice activities (C) A fast-food restaurant simulation, practicing how to count money (D) A science fair project

(A) A rubric for an oral presentation project An SLP is a Speech-Language Pathologist, one of the related-services colleagues a Special Education teacher will have the opportunity to work with. An oral presentation project is a wonderful opportunity to develop students' expressive-language skills, and this is an area that an SLP will have specific criteria they expect to see. Creating a rubric to assess the project will provide an opportunity to target specific expressive language skills using the SLP's criteria.

Which of the following transition goals meets IDEA 2004 requirements? (A) After high school, Alex will learn janitorial skills for placement in a supported employment position in the local community (B) In grade 11, Tina will pass the STAAR Alt (C) Juan will apply to community colleges and major in art (D) After high school, Mia will live at home

(A) After high school, Alex will learn janitorial skills for placement in a supported employment position in the local community. IDEA 2004 requires that transition goals be "results-oriented" and measurable. The goal should state the intended outcome for the year after the student graduates from high school. Only (A) meets all of these criteria.

What ages does an Individual Family Service Plan apply to? (A) Birth to 3 (B) 4-5 (C) 14-18 (D) 21 and older

(A) Birth to 3 An Individual Family Service Plan is provided for children birth to age 3 as part of early childhood services for children with developmental delays. At age 3, the IFSP is replaced by an IEP.

A student in an ALE classroom has autism and is nonverbal. Symbol-based communication and a picture based daily schedule are working, but progress is slow. The student has recently shown a fascination with the classroom tablet devices. What type of assistive technology might the teacher recommend the IEP committee consider for this student? (A) Communication app for the tablet device (B) Talking calculator (C) Handwriting guides (D) Speech-recognition program for the classroom computer

(A) Communication app for the tablet device An ever-increasing number of apps for tablet devices and smart phones are being developed. Many of these are useful in Special Education. For the student described in this scenario, one of the communication apps that allows teachers to build the student's visual schedule and use the voice-over capabilities of the table appears to be an ideal option to motivate the student and help him progress

A third-grade teacher displays the following words on a prominent bulletin board in her classroom: acute, right, obtuse, area, perimeter, symmetry, and volume. Which of the following instructional strategies would be the best use of the mathematical vocabulary words posted on the bulletin board? (A) Divide the students in groups of three and assign one word from the bulletin board to each group. Direct the groups to illustrate the meaning of their words on pieces of chart paper. Then have the class play a game of visual charades, where a group presents their illustration to the class, and the class guesses which vocabulary word is illustrated. (B) Avoid pointing out the words and let them be a natural part of the classroom environment for the students to absorb. (C) Have students refer to the bulletin board when completing math assignments in order to spell words correctly. (D) Have students find the same words in magazines and newspapers and staple their environmental print examples to the bulletin board.

(A) Divide the students in groups of three and assign one word from the bulletin board to each group. Direct the groups to illustrate the meaning of their words on pieces of chart paper. Then have the class play a game of visual charades, where a group presents their illustration to the class, and the class guesses which vocabulary word is illustrated. Using the words as catalysts for peer interaction and discussion of the math vocabulary allows students to internalize the meanings of the words both by visualizing them and by teaching them through the modified game of charades.

Which of the following academic difficulties may characterize a language learning disability? (A) Frequent misspelled words (B) Difficulty memorizing the multiplication tables (C) Frequent missing assignments (D) Difficulty working with other students in groups

(A) Frequent misspelled words Language-learning disabilities involve difficulties or delays with written language, grammar and spelling, and handwriting. Frequent misspelled words would be indicative of a language-learning disability.

During the first two weeks of school, a first-grade teacher observes his/her students completing various assignments and notes down their strengths and weaknesses. What kind of assessment is the teacher giving? (A) Informal (B) Norm-referenced (C) Summative (D) Formal

(A) Informal Observations are considered informal assessments.

Which of the following statements is true of autism? (A) It is more common among boys than girls. (B) It is more common among children born from teenage parents. (C) It is more common among girls than boys. (D) It is curable.

(A) It is more common among boys than girls. Autism spectrum disorders disproportionately affect boys. They are five times more common among boys (1 in 54) than among girls (1 in 252), according to the Centers for Disease Control and Prevention

A third-grade student is tested for learning disabilities and is found to have memory deficits. Based on this diagnosis, in which area of mathematics is this student likely to have difficulty? (A) Math facts (B) Geometry (C) Graphs and charts (D) Fractions, decimals, and percentages

(A) Math facts Memory deficits often result in math fact memory and retrieval issues. This makes completing math fact drills arduous for students who find themselves frustrated trying to retrieve a math fact they had memorized a few days earlier.

When writing a Behavior Intervention Plan for a student whose behavior impedes learning, what kind of strategies does IDEA 2004 require the IEP team to consider? (A) Positive behavioral interventions and supports (B) Behavior logs (C) Negative reinforcers (D) Parental strategies

(A) Positive behavioral interventions and supports IDEA 2004 specifically states that the IEP team should consider positive behavioral interventions and supports to address behaviors that impede learning. The Texas Education Agency fully supports this statement and urges schools to adopt proactive disciplinary approaches.

What details must a written notice of an ARD committee meeting include? (A) Purpose, time, and place of the meeting (B) Three choices for the date and time of the meeting (C) Evaluation results (D) The proposed IEP

(A) Purpose, time, and place of the meeting A written notice for an ARD (Admission, Review, and Dismissal) committee meeting must include the purpose, time, and place of the meeting.

As part of reading instruction, a Special Education teacher in a self-contained classroom has a student say a word, trace a word, identify a word in text, and complete a worksheet by matching the word to a picture. Which research-based instructional strategy is being demonstrated in this teacher's classroom? (A) Repetition and review (B) Computer-based instruction (C) Positive reinforcement (D) Visual teaching

(A) Repetition and review By having the student practice the word repeatedly but through different activities, the teacher makes the most of the power of repetition for students with intellectual disabilities. These students often experience retention difficulties and need strategies that provide multiple opportunities to practice without allowing the student to become bored.

Ms. Tejeda, a Special Education teacher, is assigned to co-teach several classes with an eighth-grade algebra teacher. Which of the following interactions would be the most effective way for Ms. Tejeda to ensure successful collaboration? (A) Schedule a time before or after school twice a week to plan lessons together (B) Invite the algebra teacher out for appetizers and drinks to "get to know one another" (C) Gather all the Special Education resources she uses most and drop them off on the algebra teacher's desk (D) Offer to modify all of the algebra teacher's tests for the students with special needs

(A) Schedule a time before or after school twice a week to plan lessons together When working with a coteacher, mutual lesson planning is essential to a successful collaboration. Scheduling a time to plan together twice a week and consistently being on time to these planning meetings will ensure that both teachers feel they are being respected and allowed to contribute ideas.

The first piece of federal legislation to address the civil rights of all people with disabilities was (A) Section 504 of the Rehabilitation Act. (B) Individuals with Disabilities Education Act. (C) Elementary and Secondary Education Act. (D) No Child Left Behind.

(A) Section 504 of the Rehabilitation Act, passed in 1973, was the first piece of federal legislation to address the civil rights of all people with disabilities. Previous federal legislation existed for people with learning disabilities, but not for students with all disabilities.

Which of the following is NOT a strategy that will help a teacher conduct productive and positive parent-teacher meetings? (A) State at the beginning of the meeting that the teacher has only 10 minutes and will have to end the meeting as soon as those 10 minutes are up (B) Develop a clear purpose for the meeting, state the purpose at the beginning of the meeting, and summarize how this purpose was accomplished at the end of the meeting (C) Start the meeting by noting some of the positive aspects of the child's performance and behavior (D) Schedule the meeting time around the parents' schedule needs

(A) State at the beginning of the meeting that the teacher has only 10 minutes and will have to end the meeting as soon as those 10 minutes are up Limiting a parent meeting to 10 minutes and making it clear that the meeting is not the teacher's top priority is a sure way to put parents in a defensive position. Such a short time span makes it difficult to have time for parent questions or for teacher-parent brainstorming and problem solving. When scheduling parent meetings, scheduling a minimum of 20 minutes and being available in case the meeting runs longer will go a long way toward building positive interactions.

Katie, a fourth-grade student with ADHD, is often off-task. Her regular education teacher places her at a table close to the front of the room with tablemates who are very focused on their work. What is Katie's teacher doing? (A) Ignoring Katie's behavior (B) Implementing a simple accommodation to improve Katie's performance (C) Encouraging Katie's classmates to be watchdogs (D) Modifying Katie's assignments

(B) Implementing a simple accommodation to improve Katie's performance Moving Katie closer to the front of the room and choosing tablemates who will not further distract her is a simple accommodation, which may help Katie to concentrate better on schoolwork. If this does not work, more complex accommodations would be developed.

Based on the chart, which of the following statements is true? (A) The average height of the girls in this sample is higher than the average height of the boys in this sample (B) The mode of the heights of the boys is higher than the mode of the heights of the girls (C) Fourth-grade girls are taller than fourth-grade boys (D) None of the girls in the sample were 4 foot 8 inches

(A) The average height of the girls in this sample is higher than the average height of the boys in this sample The average, or mean, is calculated by adding up all the numbers in a data set and then dividing by the amount of numbers you have. In this case, when we calculate the average heights of the girls and boys, we get 52.111 for the girls and 51.222 for the boys. Thus, the average height of the girls in the sample is higher than the average height of the boys in the sample.

In a case where the school district and the parents disagree about a student's placement in Special Education or about the services the school district will provide, either the district or the parents may call for a hearing before an impartial hearing officer. Which of the following statements regarding the hearing is true, according to Special Education laws? (A) The hearing officer must not be an employee of the school district. (B) The hearing officer will listen only to the parents and a school district representative. No one else may attend the hearing. (C) The hearing cannot be open to the public. (D) The hearing officer can only offer an opinion on the situation. It is up to the parents and the school district to resolve their own differences after the hearing concludes.

(A) The hearing officer must not be an employee of the school district. An impartial hearing officer cannot be an employee of a school district or any public agency that is involved in the education or care of the child.

Mr. Martinez, an eighth-grade social studies teacher, begins a unit on Westward Expansion in the United States by dividing students into "families" and giving each family a destination to reach in the West. Each "family" receives an index card with a list of supplies and transportation. "Family" members must talk together to figure out how they will get to their destination. What is the most likely reason Mr. Martinez begins the unit this way? (A) To build student engagement in the topic (B) To prepare students for the test (C) To foster healthy competition (D) To make decisions based on data

(A) To build student engagement in the topic Mr. Martinez uses the simulation of how it felt to be a family heading west to build anticipation and engagement in the upcoming unit.

Two days after the assessment, the Special Education teacher is with the student's class in the school library. She notices the student looking at an action/adventure book. It is approximately a 6.0 reading level. An appropriate reaction on the part of the teacher would be to (A) call the student aside and recommend a similar-themed book that is written at a 3.0 reading level. (B) let the student check out the original book. (C) walk over to the student and say, "You can't read that. It is too difficult." (D) tell the student she will use the original book for the next reading lesson they work on together.

(A) call the student aside and recommend a similar-themed book that is written at a 3.0 reading level. A book at a 6.0 reading level is in this student's frustration level based on the assessment. This means the student will likely run across a number of words he does not know how to read. This can lead to frustration and abandoning the book. Since the Special Education teacher is present in the library and sees the student choose a book on his frustration level, she is able to redirect him toward a book on his independent level. He will get far more enjoyment and reading self-confidence out of a book at his independent level.

The primary reason to provide assessment accommodations is to (A) help students with disabilities have an equal opportunity to demonstrate what they know. (B) give an advantage to students with disabilities. (C) make sure students with disabilities pass their classes. (D) make the test administrator's job more challenging.

(A) help students with disabilities have an equal opportunity to demonstrate what they know. Assessment accommodations do not give students with disabilities an advantage over other students taking the test. Instead, accommodations "level the playing field" so that students have an equal opportunity to demonstrate what they know.

An elementary ALE teacher uses puff paint and index cards to create flash cards with raised words. The words on the cards are: and, is, what, I, am, to, have, see, the, you. The teacher gives each student several minutes to trace the raised words with their fingers. If the student is verbal, the teacher has the student say the word while looking at it and tracing it. If the student is nonverbal, the teacher says the word while the student looks at the word and traces it. The most likely reason that the teacher wants the word spoken at the same time the word is being traced is to (A) increase retention through repetition and the combination of learning modalities. (B) make the activity fun. (C) make sure the words are pronounced correctly. (D) decrease disruptive behaviors.

(A) increase retention through repetition and the combination of learning modalities. By having students simultaneously look at the words, trace the raised letters, and say the word, the teacher is incorporating repetition, specifically repetition through different learning modalities: visual, tactile/kinesthetic, and auditory. This combination of repetition and multisensory learning is a powerful tool for building short- and long-term retention.

A Special Education teacher working with a student with multiple disabilities moves the student to be part of the class's circle time as part of the student's IEP. The teacher can ensure her own safety and the safety of the student during the lift and transfer by (A) lifting with her legs, not her back. (B) bending at the waist and moving the student quickly. (C) practicing with each student in her class. (D) leaving the student in his or her wheelchair, even though he or she is apart from the other students sitting on the floor for circle time.

(A) lifting with her legs, not her back. Applying knowledge of appropriate body mechanics to ensure safety in transfer, lifting, positioning, and seating is part of Competency 004. Special Education teachers receive training in the proper way to lift students. To prevent injury, it is essential to bend and lift with the legs while maintaining a straight back.

Which reauthorization of IDEA expanded the definition of disabilities to include children with developmental delays between the ages of three and nine? (A) Reauthorization of 1990 (B) Reauthorization of 1997 (C) Reauthorization of 2004 (D) Reauthorization of 2009

(B) Reauthorization of 1997 The 1997 Reauthorization of IDEA expanded the definition of disabilities to include children with developmental delays between the ages of three and nine.

Mr. Aquino, an eighth-grade reading teacher, has his students create summary notebooks. For each chapter they read, they complete a simple chart listing who the main character is, what the main character's objective is, what obstacles stand in the way of the objective, and how the character deals with the obstacle. To help students who are struggling, Mr. Aquino has the students label their charts with the words, "Who Wanted But So." This helps cue students to fill in the applicable information. Here is a sample page from one of the student's notebooks for chapter three of the book she is reading: The primary advantage of Mr. Aquino's summarization framework is to (A) prevent students from listing all the details of a story in place of a summary. (B) give students a summarization strategy they can use with all texts. (C) prevent students from cheating. (D) ensure that all students will come up with the same answer.

(A) prevent students from listing all the details of a story in place of a summary. When students are asked to summarize, they frequently make the mistake of just retelling all the high points in the story. The primary advantage of a framework like the one Mr. Aquino uses is that it gives a firm structure for students to build a summary that focuses on the gist of the story, not the details.

A fifth-grade student with learning disabilities consistently procrastinates in completing his assignments and is in danger of failing two subjects due to incomplete assignments. The student's teacher has just assigned the class a research paper that will weigh heavily on the language arts grade. An appropriate behavioral intervention for this student would be to (A) provide a daily planner and map out partial completion dates as well as the final completion date. (B) review the guidelines of the assignment and have the student sign his name, indicating he understands the due date and the consequences of not completing the assignment. (C) have the student stay in the classroom during recess every day to write the paper. (D) promise the student a candy bar for every paragraph he writes.

(A) provide a daily planner and map out partial completion dates as well as the final completion date. When a student's procrastination behaviors interfere with academic performance, the procrastination must be addressed in as proactive and positive a way as possible. Teaching a student how to use a planner and set smaller deadlines leading up to the final deadline will help the student both in school and beyond.

If a child cries easily, does not want to interact with peers, and does not like coming to school on a consistent basis, the best course of action to take would be to (A) report it to the school counselor and recommend behavioral observation by a school psychologist. (B) ignore the behaviors, as all children sometimes have issues at school. (C) seat the child with classmates who will be empathetic. (D) suggest to the parents that they should talk to their child.

(A) report it to the school counselor and recommend behavioral observation by a school psychologist. All of the listed symptoms (crying easily, avoiding interactions with peers, and not wanting to come to school) are potential symptoms of a behavioral or emotional disorder. Further observation and testing is called for in this case.

Mr. Osborn, an ALE teacher, works one-on-one with each of his students. Early in the school year, he presents each student with a list of words. He says, "I am going to show you a list of words you may or may not know. If you know the word, read it out loud. If you do not know the word, say pass." As students read the list of words, Mr. Osborn makes notes on a separate checklist. He places a check mark next to any words the student reads correctly in a half second or less. He places an "X" next to any word the student cannot read, hesitates to read, or has to work to sound out. Mr. Osborn is most likely assessing (A) students' sight-word automaticity. (B) students' grasp of the English language. (C) students' ability to follow directions. (D) students' reading comprehension.

(A) students' sight-word automaticity. Sight-word automaticity is a critical aspect of fluent reading. There are 200 frequently used words that make up over 50 percent of printed text in the English language. Many of these words do not follow typical phonetic conventions. For this reason, educators work to help students recognize these words on sight—thus the term "sight words." Automaticity is achieved when a student can call a word correctly in a half second or less.

A first-grade teacher sets up several math stations for students to rotate through in groups of four to five. The teacher sits at a large table that serves as one station and assigns students to work in pairs or groups of three to represent and solve addition problems using plastic teddy bears. As students solve the problems, the teacher observes and immediately points out mistakes so that students can try again. The teacher praises each pair or small group upon successful completion of the problem. The instructional approach that this teacher is utilizing is (A) teacher-directed cooperative learning. (B) visual discrimination. (C) generic problem solving. (D) verbalization.

(A) teacher-directed cooperative learning. Teacher-directed cooperative learning is a targeted instruction approach that involves students working in small cooperative groupings while the teacher is nearby and able to observe student strategies and provide direct and immediate feedback.

In Special Education professional circles, the term "disproportionality" refers to (A) the overrepresentation of racial, cultural, and linguistic minorities in Special Education. (B) a mathematical dilemma with geometric shapes. (C) the high number of students with autism in ALE classrooms. (D) the low number of Texas schools receiving Special Education funding.

(A) the overrepresentation of racial, cultural, and linguistic minorities in Special Education. The term "disproportionality" in Special Education refers to the overrepresentation of racial, cultural, and linguistic minorities in Special Education. This is a critical issue in Special Education and one that Special Education teachers should constantly watch for in their own evaluations and assumptions.

A seventh-grade inclusion teacher uses the interactive whiteboard to model persuasive writing for her students. She writes a persuasive essay on why school should be year round. As she writes, she comments on her word choices and talks about points to make, expanding some of them and discarding others. The instructional strategy that this teacher is using is (A) think-aloud. (B) graphic organizer. (C) language experience. (D) formula writing.

(A) think-aloud. The teacher is modeling a think-aloud, a technique where the facilitator literally thinks out loud the kinds of thoughts that writers or readers think subconsciously during the process of writing or reading. Think-alouds are great strategies for helping students realize that all writers have moments of self-doubt. They are also great opportunities to reveal how writers organize thoughts in their head before writing them down.

A Special Education teacher works with a biology teacher to create a mind map of the upcoming unit on plants. The most likely purpose for this collaboration is (A) to help both teachers identify the big ideas and how they are related to each other so that they can identify the key elements to assess as the unit progresses. (B) for the biology teacher to be able to teach the Special Education teacher science concepts. (C) to show that minds have branches and roots just like plants. (D) for the Special Education teacher to show the biology teacher a cool new technique she just learned at a staff development meeting.

(A) to help both teachers identify the big ideas and how they are related to each other so that they can identify the key elements to assess as the unit progresses. A mind map is a diagram that visually outlines key information. It is an excellent strategy for working through the key ideas of a unit of study. By collaborating on a mind map, the two teachers are figuring out the key concepts they will emphasize in instruction so that they can accurately assess student understanding during and after the unit.

Mrs. Miller, a fourth-grade teacher, asks her students to conduct a survey of heights for a sample of fourth-grade boys and girls. Mario surveys 18 students: 9 boys and 9 girls. The following chart shows his results. Based on the chart, the median height for the boys is (A) 50 inches. (B) 51 inches. (C) 52 inches. (D) 53 inches.

(B) 51 inches. A median number is the middle number in a sorted list of numbers. There are nine boys and the fifth number, directly in the middle, is 51. This is the median height.

Which of the following examples would best serve as a transition for students moving from a concrete to an abstract conceptual understanding of fractions? (A) A pie cut into eight pieces (B) A pie graph divided into four parts with one part shaded (C) An apple cut into 12 slices (D) A recipe requiring several ingredients in half and quarter cup or teaspoon measurements

(B) A pie graph divided into four parts with one part shaded The transitional stage of conceptual development involves visual or pictorial representations of objects, so a pie graph is the appropriate choice

According to the National Reading Panel's 2000 report, "Teaching Children to Read," the evidence to support phonemic awareness instruction is statistically significant. Which of the following strategies represents phonemic awareness instruction? (A) A teacher circles all the spelling errors students make in their first drafts. (B) A teacher says several rhyming words aloud and asks students what they notice about the words. She then says the word "wet" and asks students to call out words that rhyme with "wet." (C) A teacher sits beside a student who is reading out loud and models any word the student stumbles over. (D) A teacher reads a book and then asks students to point out their favorite parts.

(B) A teacher says several rhyming words aloud and asks students what they notice about the words. She then says the word "wet" and asks students to call out words that rhyme with "wet." Phonemic awareness is the ability to hear and recognize phonemes. Recognizing rhyming sounds involves recognizing the phoneme that is making the rhyme. Calling out words that rhyme with a target word demonstrates an understanding of a phoneme.

What term best describes an assessment that measures group performance against an established standard at defined points along the path toward the standard? (A) Formative (B) Benchmark (C) Summative (D) Alternative

(B) Benchmark A benchmark assessment measures group performance against an established standard at defined points.

A second-grade inclusion teacher arranges her student desks into small groups of four. She places her teacher desk on the right side of the class and an oval table on the left side. Around the oval table, she places four student chairs. She has a teacher chair on one side of the table. On the wall, she places a poster with the following words: Presenter, Researcher, Note Taker, and Discussion Leader. Given this classroom arrangement, what types of instruction would you expect to see if you visited? (A) Whole-group instruction and leveled groups (B) Cooperative learning and targeted small group instruction (C) Independent study and project-based learning (D) Flexible grouping and sustained silent reading

(B) Cooperative learning and targeted small group instruction Given the grouping of the desks and the poster specifying individual roles within each group, this classroom is organized for cooperative learning. Furthermore, the oval table on the side with four student chairs and one teacher chair indicates small-group targeted instruction.

A middle-school teacher believes in incorporating movement to help maintain student focus and engagement. Which of the following classroom situations would be an example of that philosophy? (A) Students are assigned seats and expected to stay in their seats unless they raise a hand asking for permission to sharpen a pencil or go to the restroom. (B) Each student is given an index card and asked to write a question he or she has about the lesson the teacher just presented. The teacher then has the students stand and find partners who are wearing the same color. Once all the students are standing next to their partners, the teacher asks them to exchange cards and answer each other's questions. (C) The teacher gives any student who answers a question correctly a high-five. (D) Students are asked to draw what they see in their heads as a teacher reads an article aloud to them.

(B) Each student is given an index card and asked to write a question he or she has about the lesson the teacher just presented. The teacher then has the students stand and find partners who are wearing the same color. Once all the students are standing next to their partners, the teacher asks them to exchange cards and answer each other's questions. The teacher's activity in (B) provides for purposeful movement, meeting the needs of kinesthetic learners while also focusing students on the content just taught.

What should precede the development of a Behavior Intervention Plan? (A) A tour of the student's home environment (B) Functional behavioral assessment (C) Review of the discipline policies of the student's teachers (D) Review of the student's STAAR scores

(B) Functional behavioral assessment Prior to writing a Behavior Intervention Plan, the IEP team should collect data for a functional behavioral assessment in order to identify the cause of the behavior, its frequency, and situations where the student does not exhibit the behavior. Having this data helps the team establish positive supports to redirect and reduce the inappropriate behavior.

A sixth-grade teacher hands out copies of a blank Venn diagram for students to use during prewriting. What form of writing are the students most likely going to be assigned? (A) Persuasive (B) How-to (C) Compare/contrast (D) Narrative

(C) Compare/contrast A Venn diagram is composed of two circles that intersect. It is perfect for compare/contrast writing as the comparisons can be written inside the circle while the contrasts can be written outside the circle.

A veteran teacher advises a new colleague in the Special Education department to identify some strengths she sees in each of her students and to call parents in the first few weeks of school to compliment their child on these strengths. What is the primary advantage of this approach to parent-teacher communication? (A) It allows the new teacher to ignore all the problems she is going to have with students (B) It builds respect with parents who see that the teacher cares about their child and are often pleasantly surprised to be getting a "good" call (C) It can be documented on the first evaluation the new teacher receives (D) It ensures that the new teacher will believe in herself enough to make it through the school year

(B) It builds respect with parents who see that the teacher cares about their child and are often pleasantly surprised to be getting a "good" call Positive phone calls are an excellent way to build respectful relationships with parents at the beginning of the school year. For some parents, it may be the first positive phone call they have ever received from school. By showing parents that a teacher cares about and appreciates students from the beginning, it becomes easier to problem-solve together if negative behaviors emerge later in the year.

What is likely an underlying cause of bullying behavior? (A) Racism (B) Lack of a sense of control (C) Dislike of other people (D) Overprotective parent

(B) Lack of a sense of control Exhibiting bullying behavior often stems from a deep feeling of insecurity. Rather than face his own fears, the bully makes others feel bad and exerts power to try and establish a sense of control.

A third-grade student with learning disabilities turns in the following piece of writing after 40 minutes of classroom writing time: Based on the student's writing sample, at what stage of spelling would the Special Education teacher most likely place this student? (A) Emergent (B) Letter Name-Alphabetic (C) Within-Word Pattern (D) Syllables and Affixes

(B) Letter Name-Alphabetic Based on the writing sample, the student is in the latter stages of the letter name-alphabetic stage of spelling. The student is generally starting the words with the correct sounding letter and even has some of the inner consonants and vowels correct. However, there are many interior letters missing, and all of the silent "e's" are missing.

For Special Education students with intensive disabilities, school districts must offer public education until age 23. Which of the following curriculum choices would be the most appropriate for students after high school to age 23? (A) Intensive reading intervention (B) On-the-job coaching and independent living skills (C) Academic courses along with two electives (D) Social skills training

(B) On-the-job coaching and independent living skills The ultimate goal of a posthigh school program should be to prepare students for success in their adult life in the community. For this reason, on-the-job coaching and independent living skills should be a curriculum priority.

How often must a student's transition plan be updated? (A) At the beginning of each semester (B) Once a year as part of the ARD (C) At ages 16, 18, and 21 (D) Whenever the student gets a new job experience

(B) Once a year as part of the ARD IDEA 2004 requires that a student's transition plan be updated annually along with the student's IEP. This is part of the regular ARD process in Texas.

Two co-teachers choose to use the "One Teach, One Observe" model for a lesson on volcanoes. What would this model look like in action? (A) One teacher would pretend to be a reporter on location at a volcanic eruption, reporting what she sees, while the other teacher presents the normal progression of a volcanic eruption. (B) One teacher presents the normal progression of a volcanic eruption, using visuals and 3-D models, while the other teacher watches the students in the class and keeps notes on their levels of participation and conceptual understanding. (C) One teacher presents information to the boys while the other teacher presents the same information to the girls. (D) One teacher sets up a station with a model of an erupting volcano and the other teacher sets up an interactive whiteboard activity that involves labeling the parts of a volcano. Students rotate between the stations.

(B) One teacher presents the normal progression of a volcanic eruption, using visuals and 3-D models, while the other teacher watches the students in the class and keeps notes on their levels of participation and conceptual understanding. The "One Teach, One Observe" co-teaching model is ideal for gathering informal observational data on students. While one teacher presents the lesson, the other teacher circulates around the room and makes notes about students' behavior, level of participation, and level of conceptual understanding.

What type of shape is shown below?

(B) Parallelogram A parallelogram is a four-sided shape with two pairs of parallel sides. The facing sides of a parallelogram are of equal length and the opposite angles are of equal measure.

Why were the Rehabilitation Act of 1973 and Public Law 94-142, passed by Congress in 1975, so important for students with disabilities in the United States? (A) These two laws showed the rest of the world that the United States was committed to Special Education (B) Prior to these two pieces of federal legislation, individual states were not offering public education to the majority of children with disabilities (C) These two laws created a designated day to honor individuals with disabilities (D) These two laws made provisions for the federal government to fund over half of the Special Education expenses incurred by school districts

(B) Prior to these two pieces of federal legislation, individual states were not offering public education to the majority of children with disabilities Prior to the Rehabilitation Act of 1973 and Public Law 94-142 (later named Individuals with Disabilities Education Act), the number of students with disabilities receiving free public education was quite low across the United States. These two pieces of federal legislation opened educational opportunities for all students with disabilities by requiring states to offer a free and appropriate public education to all students.

In assessing a sixth-grade student's oral reading, a Special Education teacher uses several fictional passages leveled from 1.0 to 6.0. The teacher places each passage in front of the student, starting with the 1.0 passage, and asks the student to read it aloud. The chart below represents the results of the assessment: Based on the results, what level text would be the most appropriate to use for guided reading instruction? (A) Reading Level 2.0 (B) Reading Level 4.0 (C) Reading Level 5.0 (D) Reading Level 6.0

(B) Reading Level 4.0 A student's instructional reading level is considered to be at 90 percent accuracy or one mistake out of every 10 words; 92 percent accuracy is the closest number to 90 percent and falls into the instructional to independent reading range.

An elementary ALE teacher uses puff paint and index cards to create flash cards with raised words. The words on the cards are: and, is, what, I, am, to, have, see, the, you. The teacher gives each student several minutes to trace the raised words with their fingers. If the student is verbal, the teacher has the student say the word while looking at it and tracing it. If the student is nonverbal, the teacher says the word while the student looks at the word and traces it. What kind of words has the teacher selected for this activity? (A) Second-grade words (B) Sight words (C) Decodable words (D) Academic vocabulary

(B) Sight words All of the words listed are sight words. In fact, all 10 are from the Dolch sight-word list

Which of the following skills would be an example of a child's gross motor capabilities? (A) Drawing with a crayon (B) Skipping along a sidewalk (C) Talking to a friend (D) Solving an addition problem

(B) Skipping along a sidewalk Gross motor skills involve large body movements. Skipping is a gross motor ability.

An initial ARD is held for a student evaluated for Special Education services. The student's parents gave permission for the evaluation, but during the ARD committee meeting, they talk about their concerns regarding Special Education labels and ultimately refuse to sign their agreement to the Special Education placement. The classroom teacher feels strongly that the child needs Special Education services and requests to have a Special Education co-teacher work with the student one-on-one "off-therecord." What is the appropriate response to this suggestion? (A) The classroom teacher should be fired for violating the Texas Code of Ethics for Educators. (B) Special Education laws require parental consent. The student must be served in the regular classroom for now. (C) The classroom teacher should call the parents and let them know her plans. (D) The school district will file an appeal to an impartial hearing officer and. In the meantime, the child will receive Special Education services per the teacher's request.

(B) Special Education laws require parental consent. The student must be served in the regular classroom for now. Parental consent must be obtained before a student with a disability can be placed in a school's Special Education program. If parental consent cannot be obtained, a school district may appeal to an impartial hearing officer. A classroom teacher cannot make a decision to ignore this requirement. In the example above, the student must be served in the regular classroom for now.

Which of the following situations would be a good reason to provide community-based instruction? (A) Ninth-grade students do not see how Algebra relates to the real world (B) Students in a high school ALE class have been practicing how to order food off a menu at a restaurant (C) Students in a ninth-grade English class all passed the STAAR (D) Parents want to know why the district chose a particular reading program

(B) Students in a high school ALE class have been practicing how to order food off a menu at a restaurant Community-based instruction is frequently used in self-contained classrooms as part of preparing students to transition into the community. It makes sense to have an Alternative Learning Environment class (that has been practicing ordering food off a menu) actually go to a restaurant and order off a menu. This allows teachers to assess generalization of skills.

Mr. Garcia has his fourth-grade students read to themselves the following sentence from a Texas history text: "Juneteenth is the oldest known celebration commemorating the ending of slavery in the United States." Mr. Garcia then asks the students what the term "commemorating" means in the sentence. He suggests they look at other words in the sentence to help them find the meaning. What reading comprehension strategy is Mr. Garcia teaching his students to use? (A) Identifying the main idea (B) Using context clues (C) Visualizing (D) Blending

(B) Using context clues Context clues are the words around an unknown word in a text. By having students find surrounding words that can help them determine the likely meaning of "commemorating," Mr. Garcia is building students' independent reading strategies.

An elementary school hosts an open house a few days before school starts so that families can meet the teachers. Parents of a new student with autism come to the open house out of breath and scowling. They look around the classroom, and the mother immediately begins asking about types of sensory equipment she would like to see in the classroom. The father interrupts and says the student needs to be treated like every other student with no special equipment. The best way for the teacher to approach these parents is to (A) assume they are going to be difficult and maintain a professional but distant approach. (B) approach both parents with a warm smile and excitement about the goals for the upcoming school year, and encourage the student to explore the classroom while asking the parents to talk about their son's strengths. (C) address the mother and promise to ask the principal to purchase the sensory equipment. (D) address the father and assure him that the son will have access to the standard state curriculum.

(B) approach both parents with a warm smile and excitement about the goals for the upcoming school year, and encourage the student to explore the classroom while asking the parents to talk about their son's strengths. Working with families of children with disabilities requires empathy and tact. Many families experience conflict as parents struggle to understand and accept their child's learning difficulties. Teachers play a critical role in helping families access information, listening to families' concerns, and keeping the focus on the child and what is best for her or him. In this example, the teacher should maintain a friendly demeanor and an excitement about the opportunity to work with the child. Refocusing the parents on discussing the child's strengths will provide the teacher with valuable information and turn the event into a more positive outing for everyone.

Mrs. Seville, a seventh-grade mathematics teacher, creates an end-of-unit test for her geometry unit. All of the questions are open-ended and require students to show their work and write their answers. A Special Education co-teacher might modify this test for students with learning disabilities in Mrs. Seville's class by (A) making no modifications; all students must take the same test to be fair. (B) converting most of the questions to multiple choice, but still require students to show their work on the paper. (C) reading the test questions out loud for students. (D) providing a calculator.

(B) converting most of the questions to multiple choice, but still require students to show their work on the paper. Students with learning disabilities, particularly language-learning disabilities, will be at a disadvantage with an all open-ended, short-answer assessment. Modifying the questions by making them multiple choice is an appropriate modification. The students are still expected to do the math and show their work; they are just not tasked with composing complete sentences.

The purpose of a standardized achievement test is to (A) identify the best reading method to use with a student. (B) measure the amount of knowledge and skills a student has acquired in comparison to a larger group. (C) establish a baseline for learning prior to starting a new unit. (D) help a teacher form instructional groups.

(B) measure the amount of knowledge and skills a student has acquired in comparison to a larger group. Standardized achievement tests score students in comparison to a larger group. These tests have gone through a series of tests to ensure they are as statistically fair as possible

A fourth-grade teacher has a class of 22 students, including four students with IEPs and two English learners. At the beginning of the school year, the teacher administers an inventory with questions about how students approach different learning tasks and what subjects they like best. She also structures a variety of activities, including a memory game similar to Concentration, a classroom scavenger hunt for clues to the answer to a question, and a math game with manipulatives. As students complete the activities, the teacher observes and makes notes on a clipboard. The teacher is most likely (A) figuring out who the smartest students are. (B) observing and noting each student's learning strengths so that she can plan appropriate activities. (C) grouping students by ability. (D) making sure every student likes her.

(B) observing and noting each student's learning strengths so that she can plan appropriate activities. The teacher is providing learning activities that cover all of the major modalities: visual, auditory, tactile, and kinesthetic. Between her observations during the activities and the students' answers to the informal inventory, she will have a good grasp of each student's learning strengths so she can plan instruction accordingly

As part of the morning circle time in an elementary ALE classroom, the teacher has a small pocket chart with three pockets labeled "hundreds," "tens," and "ones." Each day, a student is selected to place a yellow straw in the "ones" pocket and then count all of the straws in that pocket. If there are 10 straws, the student gets to exchange them for a blue straw, which goes in the "tens" pocket. The mathematical concept the teacher is using to count the days of the school year is (A) number sense. (B) place value. (C) patterns. (D) fractions.

(B) place value. Any time students are working with "hundreds," "tens," and "ones," they are focused on place value. This example also shows how a concept such as place value can be integrated into a regular routine like circle time and made concrete.

One week before a geography teacher begins the next unit in the textbook, her co-teacher conducts an activity where he provides each student with half an index card with part of a word on it. Students have to stand and find the student who has the other half of their card. Then they work with their partner to find the definition for the vocabulary word on their card and draw a picture of what the word represents. The purpose of this lesson is to (A) give students a break from the textbook. (B) preteach vocabulary and thus improve students' ability to read and understand the next chapter. (C) show that all geography-related vocabulary words make great pictures. (D) give the co-teacher a chance to conduct a lesson.

(B) preteach vocabulary and thus improve students' ability to read and understand the next chapter. Preteaching vocabulary is a research-based strategy for improving reading comprehension. In a content-area class such as geography, students encounter a number of academic vocabulary words that they must understand to grasp central concepts. A hands-on vocabulary strategy like the one described helps students take ownership of the words they will encounter in the unit.

A fourth-grade teacher assigns several three-digit multiplication problems for her students to complete. In reviewing the papers, the teacher finds one with over 50 percent of the answers incorrect. Attempting to see what caused the mistakes, the teacher notices that the student has misaligned many of the numbers. The computation that was done is correct, but the wrong numbers were often multiplied or added together. In reflecting on this student's past math performance, the teacher recalls that geometry also proved difficult for the student. The assumption that would most likely be true about this student is that the student (A) hates math and does not put forth effort to do the work neatly. (B) seems to be exhibiting visual-spatial deficits that are affecting academic performance. (C) has Attention Deficit Hyperactivity Disorder. (D) requires handwriting supports.

(B) seems to be exhibiting visual-spatial deficits that are affecting academic performance. Visual-spatial deficits cause difficulty with conceptualizing geometric figures and aligning numbers. Based on the teacher's observations of this student, visual spatial deficits seem likely.

A middle-school teacher is working with a small group of belowlevel readers. He brings in five print advertisements for various consumer products and uses sticky notes to cover all the words. He then displays the advertisements one at a time and asks the students to tell him what each advertisement is trying to tell consumers. The purpose of this activity is to (A) convince the students to buy the products. (B) show students how visual images in media communicate a message and how to interpret them. (C) motivate reluctant readers by removing all the words. (D) help students visualize while reading.

(B) show students how visual images in media communicate a message and how to interpret them. Messages, particularly in our media-rich world, are conveyed with more than just words. Helping students understand this concept through advertisements that have had their words removed or hidden helps students become more critical readers and viewers. It also helps struggling readers learn that they can communicate with more than just words.

At what point should a Special Education teacher in Texas advise parents of a child with intellectual disabilities to apply for state assistance programs (for example, Community Living Assistance and Support Services)? (A) At the point when Special Education testing begins (B) When the student graduates from high school (C) As soon as the student has an official diagnosis (D) When parents can no longer care for their child

(C) As soon as the student has an official diagnosis Many of the state assistance programs for individuals with disabilities have waiting lists that can last for years. However, these lists also require an official diagnosis as a qualifier. Advise parents to call the appropriate agencies and get on the waiting lists as soon as a diagnosis is assigned.

What is the most commonly diagnosed behavioral disorder in children? (A) Autism (B) Depression (C) Attention deficit hyperactivity disorder (D) Oppositional disorder

(C) Attention deficit hyperactivity disorder Attention deficit hyperactivity disorders are the most commonly diagnosed behavioral disorders in children, affecting 3 to 6 percent.

Mrs. Benson, a third-grade teacher, has a diverse class that includes students with learning disabilities, English learners, and gifted students. For a reading unit, she pairs the students based on personality (for example, patient fluent readers with struggling readers). She then has the partners complete together every reading assignment in that unit. For text reading, partners decide how they will divide up the text for reading (for example, switching off by paragraphs or pages). For comprehension activities, partners turn in one paper representing work from both students. While partners are working together, Mrs. Benson walks around the classroom observing pairs and assisting with questions. One of the primary advantages of Mrs. Benson's paired reading strategy is that (A) struggling students can let their partners do all their work. (B) students get a great deal more supported oral reading practice than they would in a group-reading situation. (C) Mrs. Benson can take a break from lesson planning. (D) students are not paired with their friends so there is less off task talking.

(B) students get a great deal more supported oral reading practice than they would in a group-reading situation. Paired reading is an excellent method of providing peer support for reading assignments while freeing the teacher to monitor and work with pairs as needed. In a small group setting, a teacher has to split instructional time as evenly as possible. In paired reading, every student has a partner to read to, and this facilitates more supported oral reading practice than students would receive in a group setting.

A five-year-old child and his parents have just moved into the school district's boundaries. The parents bring the child to the school to register for kindergarten and explain that he is not yet speaking. The school psychologist immediately begins the paperwork to refer the child for Special Education evaluation. In selecting assessments for this child to complete, the psychologist and test administrators must keep in mind that (A) only paper-and-pencil tests are accepted in determining eligibility. (B) tests must be selected and administered so that the child's inability to speak does not interfere with the assessment of the child's aptitude or achievement level, if at all possible. (C) there must be an interpreter present during testing who can communicate with the child using sign language. (D) the child is only five and will likely learn to speak eventually.

(B) tests must be selected and administered so that the child's inability to speak does not interfere with the assessment of the child's aptitude or achievement level, if at all possible. When administering diagnostic assessments for Special Education eligibility, diagnosticians and school psychologists must seek out assessments that ensure a student's disability or first language do not interfere with the assessment of the student's aptitude or achievement level where possible. For a child who does not speak, this means that the diagnostic tests must give the child an opportunity to demonstrate aptitude without the necessity of a verbal response.

Mrs. Miller, a fourth-grade teacher, asks her students to conduct a survey of heights for a sample of fourth-grade boys and girls. Mario surveys 18 students: 9 boys and 9 girls. The following chart shows his results. Based on the chart, the range of heights for the girls is (A) 9 inches. (B) 10 inches. (C) 11 inches. (D) 47 inches.

(C) 11 inches. The range is calculated by finding the difference between the largest number in a data set and the smallest number. For the girls, the largest number is 58 and the smallest number is 47, so if you subtract 47 from 58 (58 - 47 = 11), the range is 11 inches.

At what age does IDEA 2004 say a transition plan MUST be in place? (A) 14 (B) 15 (C) 16 (D) 18

(C) 16 IDEA 2004 states that a transition plan must be in place by age 16. Many people feel that is too late to appropriately plan for transition, and in 2011, the Texas legislature made 14 the required age for transition planning in Texas.

Given the algebraic equation 3x + 4x - 1 = 34, the value of x is (A) 3. (B) 4. (C) 5. (D) 10.

(C) 5. To solve for x in (3x + 4x - 1 = 34), the first step would be to subtract 1 from the left side of the equation and add it to the right side of the equation, giving us (3x + 4x = 35). We then add 3x and 4x on the left side of the equation to get (7x = 35). We then divide both sides by 7 to get (x = 5).

Look at the following test results. The percentile score means that (A) the student passed the test. (B) the student failed the test. (C) 65 percent of students who took the test scored equal to or less than the test-taker. (D) 35 percent of students who took the test scored less than the test-taker.

(C) 65 percent of students who took the test scored equal to or less than the test-taker. A percentile rank indicates the percentage of a reference or norm group obtaining scores equal to or less than the test-taker's score.

Research on effective math instructional strategies has indicated that explicit instruction is critical. Which of the following lessons is an example of an explicit teaching strategy? (A) A teacher has students complete worksheets with operational drills for 20 minutes every day (B) A teacher gives students 20 colored squares and then steps back and observes what the students do with the squares (C) A teacher demonstrates how to solve an addition problem using a number line, talking out loud through each step of the addition process (D) A teacher assigns students homework over the weekend, telling them to find examples of math in the world around them

(C) A teacher demonstrates how to solve an addition problem using a number line, talking out loud through each step of the addition process Explicit instruction involves helping students see how a math strategy is applied step-by-step. Modeling and verbalizing each step gives students a concrete picture of how to solve a problem.

Mr. Aquino, an eighth-grade reading teacher, has his students create summary notebooks. For each chapter they read, they complete a simple chart listing who the main character is, what the main character's objective is, what obstacles stand in the way of the objective, and how the character deals with the obstacle. To help students who are struggling, Mr. Aquino has the students label their charts with the words, "Who Wanted But So." This helps cue students to fill in the applicable information. Here is a sample page from one of the student's notebooks for chapter three of the book she is reading: Which stage of the reading process is Mr. Aquino's strategy designed for? (A) Prereading (B) During reading (C) After reading (D) Emergent reading

(C) After reading Summarization is a strategy used after reading to pinpoint the gist of the storyline in a narrative text.

Which of the following is NOT an area in which diagnostic assessments to determine eligibility for Special Education services might be given? (A) Development (B) Social emotional behavior (C) Career placement (D) Auditory processing

(C) Career placement When determining eligibility for Special Education services,

Which of the following is NOT a stage of the writing process? (A) Editing (B) Prewriting (C) Counting the words (D) Publishing

(C) Counting the words Most educators think of writing as a five-step process: prewriting, drafting, revising, editing (proofreading), and publishing. Counting the words is not a part of this process, although writers are often curious to know how many words they have written.

Mrs. Cullen is a sixth-grade reading teacher. She has both regular and Special Education students in her classroom. At the beginning of the year, she administers a variety of informal assessments and observes students' strengths and weaknesses. She then sets up stations around the classroom where students can work independently or in pairs on reading and writing tasks. Each day, for 15 minutes, Mrs. Cullen teaches a whole group mini-lesson related to the TEKS. Then students are directed to stations. During station time, Mrs. Cullen conducts small-group targeted lessons on various reading skills. While the groups never have more than five students, the students called up each day change depending on the skill being taught. Based on the description of Mrs. Cullen's beginning-of-the-year procedures, which of the following statements most likely describes her philosophy? (A) Special Education students need to be taught differently (B) Teaching to the state standardized test ensures all students will pass (C) Data should drive instruction (D) There is only one way to teach reading

(C) Data should drive instruction Mrs. Cullen's decision to administer a variety of informal assessments at the beginning of the year shows that she values data when planning instruction.

What is the ARD committee in Texas equivalent to in federal regulations regarding Special Education evaluation procedures? (A) FAPE committee (B) LRE committee (C) IEP committee (D) CEC committee

(C) IEP committee The ARD (Admission, Review, and Dismissal) committee is the Texas equivalent to the IEP committee referenced in IDEA regulations.

Which of the following student activities is an example of structural analysis? (A) Matching words to pictures (B) Reading a 100-word passage in less than a minute (C) Identifying prefixes and suffixes in a list of words (D) Using a graphic organizer for prewriting

(C) Identifying prefixes and suffixes in a list of words Structural analysis involves seeing parts of multisyllabic words in order to decode them. Prefixes and suffixes are word parts that carry a predictable meaning. Being able to see them in long words also means being able to guess at the meaning of those words

A parent tells you her/his child has dyscalculia. In which academic area would you anticipate the student to have difficulties? (A) Reading (B) Writing (C) Mathematics (D) Social Studies

(C) Mathematics "Dyscalculia" means a problem with making calculations. It is a common term for a mathematics learning disability.

A third-grade teacher displays the following words on a prominent bulletin board in her classroom: acute, right, obtuse, area, perimeter, symmetry, and volume. Given these vocabulary words, what kind of math unit is the teacher most likely preparing to present? (A) Geometry (B) Probability and statistics (C) Measurement (D) Number sense

(C) Measurement The words displayed on the bulletin board are all math vocabulary related to measurement. The teacher is preparing students for a unit on measurement.

Mr. Boychuk is a seventh-grade mathematics teacher. He is assigned a co-teacher because he has several students with IEPs in his classes. Mr. Boychuk provides step-by-step explanations for each mathematical concept he presents. The co-teacher volunteers to illustrate the explanations on the interactive whiteboard while Mr. Boychuk gives the lesson. After the lesson, the co-teacher pulls together small groups and provides manipulatives. She then asks the students to use the manipulatives to demonstrate the steps Mr. Boychuk explained. She encourages students to narrate the steps as they demonstrate them. What instructional strategy is the co-teacher using? (A) Simulation (B) Game-based teaching (C) Multisensory approach (D) Visualization

(C) Multisensory approach By providing visual reinforcements and concrete examples, the co-teacher is providing multiple ways for students to grasp the instructional concepts.

Which of the following forms of communication would be considered two-way communication? (A) Weekly school newsletters (B) Email blasts from the school principal (C) Parent-teacher phone call (D) Report card

(C) Parent-teacher phone call A parent-teacher phone call is an example of two-way communication because both the parent and the teacher have an opportunity to speak and to listen.

A Special Education co-teacher is working with a fifth-grade student with learning disabilities. As the student works through a series of math problems, the teacher observes that word problems, particularly ones with multiple steps, seem particularly challenging for the student. The student often does just the first operation he can figure out and then abandons the problem. Based on this informal assessment, what type of cognitive deficits is the student most likely exhibiting? (A) Visual-spatial deficits (B) Memory deficits (C) Procedural learning deficits (D) Number sense deficits

(C) Procedural learning deficits Students with procedural learning deficits struggle to apply adequate sequencing when attempting to solve multistep computations or word problems. They are often unsure of which operation to use first or how to structure a problem. It is common for students with procedural learning deficits to use immature strategies like this student does when he completes the first operation and then abandons any further steps.

The law that mandates that states provide a free appropriate public education to all children with disabilities or risk educational funding cuts or withholdings is (A) Public Law 89-10. (B) Public Law 93-112. (C) Public Law 94-142. (D) Public Law 94-479.

(C) Public Law 94-142. Public Law 94-142, better known as IDEA, mandates a free appropriate public education for all children with disabilities. States that do not adhere to these mandates risk educational funding cuts or witholdings.

Which reauthorization of IDEA implemented a timeframe of not more than 60 days to conduct initial evaluations for Special Education eligibility? (A) Reauthorization of 1990 (B) Reauthorization of 1997 (C) Reauthorization of 2004 (D) Reauthorization of 2009

(C) Reauthorization of 2004 The IDEA Reauthorization of 2004 includes a timeline of 60 days to conduct initial evaluations for Special Education eligibility.

A test administrator, per a student's IEP, administers the test in a location with minimal distractions. What type of assessment accommodation would this be? (A) Response (B) Presentation (C) Setting (D) Timing

(C) Setting Administering the test in a place with minimal distractions is an accommodation in the setting or location.

Parents of a child with math learning disabilities want tips on how to help their child with homework. Which of the following Special Education journals would have articles that would be most appropriate to share with these parents? (A) Exceptional Children from CEC (B) Educational Researcher from AERA (C) TEACHING Exceptional Children from CEC (D) The Examiner newsletter from IDA

(C) TEACHING Exceptional Children from CEC TEACHING Exceptional Children from the Council for Exceptional Children is a practical journal that features research-to-practice information and materials for classroom use. Parents will find articles fairly easy to navigate, and several articles are specific to math strategies.

To fulfill the IDEA requirement of least restrictive environment (LRE), which of the following areas must be documented in a Special Education student's IEP? (A) The name of the student's Special Education teacher (B) The list of medications the student is taking (C) The extent to which the student will not participate with nondisabled students in regular classes or nonacademic activities (D) The parents' description of the student's home environment

(C) The extent to which the student will not participate with nondisabled students in regular classes or nonacademic activities. The extent to which a student will not participate with nondisabled students in regular classes or nonacademic activities must be clearly documented in the student's IEP. This documentation is necessary to ensure that the ARD committee has addressed opportunities for inclusion and has documented decisions to limit inclusion for educationally sound reasons. This documentation is all part of fulfilling the least restrictive environment requirement of IDEA.

A second grader with autism who is nonverbal gets upset in the cafeteria at lunch and lies down on the ground and begins screaming. This goes on for several minutes, disrupting lunch. Which of the following is NOT a potential cause of this behavior? (A) The noise and confusion in the cafeteria became overwhelming. (B) The child's scheduled lunchtime is normally an hour later and today is a change of routine. (C) The second grader wants everyone to look at him. (D) The child's favorite type of chip was not in his lunch bag today.

(C) The second grader wants everyone to look at him. Individuals with autism lack "theory of mind," the ability to recognize that other people have different perspectives. They do not do tantrums because they want to be the center of attention or want to manipulate others. Instead, they act out in frustration because they cannot communicate appropriately how they are feeling, and their frustration builds to the breaking point.

Which of the following choices is NOT one of the purposes of a transition assessment? (A) To identify a student's interests and vocational abilities (B) To identify a student's level of self-determination skills (C) To determine whether a student will pass high school (D) To identify a student's strengths, abilities, and potential deficits

(C) To determine whether a student will pass high school A transition assessment is built to help educators guide students to post-school jobs and activities that are likely to be most fulfilling to them. It is not used to determine whether a student graduates from high school or not.

A Special Education teacher has two paraprofessionals assigned to her class. Which of the following would be the best use of these assistants' time? (A) Have the paraprofessionals make copies and laminate materials most of the day (B) Ask the paraprofessionals to administer medications to the students (C) Train the paraprofessionals to help facilitate the instructional programs the teacher is using so that instruction can be divided into individual and small group time (D) Have the paraprofessionals monitor the students while the teacher does required paperwork

(C) Train the paraprofessionals to help facilitate the instructional programs the teacher is using so that instruction can be divided into individual and small group time Special Education classrooms are meant to support individualized and targeted instruction. Paraprofessionals can be excellent instructional resources when trained in the programs students are using.

A grade 4 student with learning disabilities is struggling with language arts. The student likes to compose stories, but her handwriting is extremely difficult to read and she makes numerous spelling errors. What assistive technology would you recommend the child be evaluated for? (A) Recorded books (B) Alternative keyboard (C) Word processor with a spell checker (D) Electronic math worksheets

(C) Word processor with a spell checker Based on the description, the student's primary difficulty with writing appears to be fine motor (poor handwriting) and lack of the alphabetic principle or difficulty with memory (poor spelling). While Special Education testing can give a much better picture of the causes, a simple accommodation such as providing access to a computer or word processor (where the student can type her papers and use a spell checker to identify misspelled words and correct them) can be provided with or without a Special Education designation.

A Special Education teacher has several students who speak Spanish at home, but the teacher is not bilingual. The best approach for this teacher to take when communicating with Spanish-speaking parents is to (A) send home written notes in English and instruct students to translate the notes for their parents. (B) talk to parents using a lot of gestures to communicate meaning. (C) arrange for a bilingual colleague to serve as an interpreter during all parent phone calls and parent-teacher conferences. (D) send all communication via emails so the parents can use online translation programs.

(C) arrange for a bilingual colleague to serve as an interpreter during all parent phone calls and parent-teacher conferences. When working with parents who do not speak English, it is imperative to find a translator at the school or via a translation phone service to ensure that teachers and parents can truly communicate. Making sure to have the translator at all meetings is a way for the teacher to show her respect for the parents and the language they speak.

A Special Education teacher working with students with intellectual disabilities and autism is teaching one-to-one correspondence. The most appropriate instructional tool for the teacher to use in teaching this concept would be (A) a worksheet with symbol-supported text. (B) a graphic organizer. (C) blocks of different colors. (D) a calculator.

(C) blocks of different colors. Students with intellectual disabilities need concepts to be as concrete as possible. Providing manipulatives like colored blocks allows students to place each block as they identify it by number (for example, one block, two blocks).

The best way for dealing with a student who acts silly and talks loudly throughout class time is to (A) send the student to time out. (B) ignore the behavior. (C) create a leadership role for this student. (D) make sure all the other students in the class see you reprimanding the student.

(C) create a leadership role for this student. Acting silly and talking loudly are attention-seeking behaviors. This is a student who wants to be noticed. By assigning a leadership role in the class to this student, the teacher is providing an appropriate way for the student to be recognized.

A middle-school Special Education teacher is teaching a mathematics class to seventh graders who failed the sixth-grade STAAR math test. The teacher provides highlighters to each student and shows them a list of keywords to look for and highlight in a series of word problems. Each time the students are assigned word problems, the teacher has them highlight keywords. She tells her students that this technique will help them on the test. The strategy that the teacher is providing her students is (A) pictorial representation. (B) corrective feedback. (C) generic problem-solving strategy. (D) modeling strategy.

(C) generic problem-solving strategy. Teaching students to highlight keywords every time they encounter a word problem is a generic problem-solving strategy that can help struggling learners slow down and think about the clues the words provide for solving the problem

In order to build student's fluency, an appropriate prereading strategy would be to (A) have students stop at the end of each page and summarize what happened. (B) have students turn to a partner and talk about what they liked and didn't like in the story. (C) have students scan the text and then predict what the text will be about. (D) have the students look at the title and then name similar titles from other books.

(C) have students scan the text and then predict what the text will be about. The act of scanning or skimming a text allows a reader to get a sense of the topic and words before actually reading. Students use this knowledge to make their predictions and build interest in the text. This prereading strategy builds fluency by helping students feel more prepared for the words they encounter.

The purpose of a transition plan in a student's IEP is to (A) help the student move from elementary to middle school. (B) help the student move from middle school into high school. (C) help the student move from high school to postgraduation and adult life. (D) help students move from Special Education to general education.

(C) help the student move from high school to postgraduation and adult life. By IDEA definition, the purpose of a transition plan is to help a student successfully move from high school to postgraduation and adult life

A Special Education teacher is working with a small group of students with documented social-skills goals in their IEPs. An appropriate activity for the small group to do would be (A) sustained silent reading. (B) taking notes as the teacher talks about the benefits of good social skills. (C) playing a board game built around conversation skills. (D) writing goals about how many friends each student wants to make in the next month.

(C) playing a board game built around conversation skills. Improving social skills involves setting up scenarios where students can practice interacting with others. A board game focused on conversation skills provides an activity that encourages students to talk to each other and use questions and comments to carry on a conversation.

When planning instruction for students with special needs, the best approach is to (A) focus on students' weaknesses. (B) follow the lessons in textbooks and programs exactly. (C) teach to students' strengths. (D) teach all students the same way.

(C) teach to students' strengths. Teaching to a student's strengths will ensure both more rapid success and increased self esteem.

A middle-school Special Education teacher reviews a student's grades as part of planning her IEP goals for the next ARD. She sees the following: When the teacher reflects on the general education teachers the student has, she knows that the Reading teacher incorporates a lot of movement and critical thinking activities into her lesson plans. The language arts teacher requires students to write a paper every week as homework. The social studies teacher assigns chapters and gives open-ended unit tests. The science teacher has students conduct at least two experiments per week. The teacher might conclude that (A) the student wants to be an astronaut. (B) the student only likes PE. (C) the student may be a tactile/kinesthetic learner and needs some explicit reading and writing strategy instruction. (D) the student may be an auditory learner and clearly does not like her social studies and language arts teachers.

(C) the student may be a tactile/kinesthetic learner and needs some explicit reading and writing strategy instruction. The student's best grade is PE, and her second-best grade is in a class that emphasizes hands-on experiences; so there are a couple of indications that she may be a tactile/kinesthetic learner. The Special Education teacher would have to confirm that through observation and learning styles inventories. The low grades in the content areas of language arts and social studies are a tip-off to the student's struggles with reading and writing. The fact that the reading teacher incorporates movement into her lesson plans may be part of the reason the reading grade is passing.

A container of milk is 3/8 full. What percentage of the container is full? (A) 0.5 (B) 0.38 (C) 0.75 (D) 0.375

(D) 0.375 3/8 converts to 0.375 in decimal form. This can be found by dividing 3 by 8 on a calculator.

What techniques may be used to conduct a functional behavioral assessment? (A) Data analysis (B) Structured interviews with current teachers (C) Standardized behavior assessment (D) All of the above

(D) All of the above When conducting a Functional Behavioral Assessment, data analysis, indirect assessment (including structured interviews), and direct assessment (including standardized assessments) may all be used. However, they are not all required. It is up to the IEP team what techniques they will use to gather the information they need to identify the root cause of the inappropriate behavior. Competency 006

When communicating with parents, what is the best form of communication in today's world? (A) Emails (B) Texts (C) Voice mail (D) Any of the above, depending on the parents' preferences

(D) Any of the above, depending on the parents' preferences Mobile technologies have expanded the forms of communication teachers can use and make communication more immediate in many cases. However, the key to positive parent communication is to find out how parents prefer to communicate. Some may have busy careers and not appreciate a phone call on their cell phone during work hours. Others may not be comfortable with texting. At the beginning of the year, teachers can establish positive two-way communication channels by asking parents to indicate their preferred method of communication.

Ms. Silva, a co-teacher in a second-grade classroom, works with a group of three students. She reads a sentence out loud and then has the students repeat the sentence. She instructs them to "mimic" her as closely as possible because she wants them to match her speed and expression. What modeling method is Ms. Silva using? (A) Recorded books (B) Neurological impress (C) Choral reading (D) Echo reading

(D) Echo reading Echo reading is a modeling method in which a fluent reader takes the lead and reads a sentence or line from a poem and then a developing reader repeats the sentence or line, using the same expression and speed if possible. Like all modeling methods, it provides a level of security for a struggling reader because the fluent reader pronounces all the words before the struggling student has to do so.

Mrs. Cullen is a sixth-grade reading teacher. She has both regular and Special Education students in her classroom. At the beginning of the year, she administers a variety of informal assessments and observes students' strengths and weaknesses. She then sets up stations around the classroom where students can work independently or in pairs on reading and writing tasks. Each day, for 15 minutes, Mrs. Cullen teaches a whole group mini-lesson related to the TEKS. Then students are directed to stations. During station time, Mrs. Cullen conducts small-group targeted lessons on various reading skills. While the groups never have more than five students, the students called up each day change depending on the skill being taught. What instructional technique is Mrs. Cullen using? (A) Homogenous grouping (B) Whole-group instruction (C) Self-selected reading (D) Flexible grouping

(D) Flexible grouping Mrs. Cullen is using flexible grouping to ensure that students receive targeted instruction in the skill areas they most need to improve.

A teacher presents students with the following number pattern: 1, 3, 9, 27, What type of number pattern is this? (A) Arithmetic sequence (B) Fibonacci sequence (C) Harmonic sequence (D) Geometric sequence

(D) Geometric sequence A geometric sequence is made by multiplying by a given value each time. This number sequence has a factor of 3. The pattern is continued by multiplying by 3 each time.

Parents of a fifth grader with learning disabilities have a parentteacher conference with the student's classroom teacher and Special Education co-teacher. The parents indicate that they would like to learn more about learning disabilities and the potential schools and camps that may be appropriate for their child. An appropriate resource for the teachers to share with the parents would be information about (A) local camps for children with autism. (B) local group homes and other residential facilities. (C) the location of local libraries. (D) LDA and a link to the organization's website.

(D) LDA and a link to the organization's website. LDA is the Learning Disabilities Association of America. It is an organization of parents and teachers of children with disabilities. Many of its members are individuals with learning disabilities who advocate for themselves. The organization's website is www.ldanatl.org. Providing parents with this information and link will give them information about learning disabilities and about a wide variety of schools and camps dedicated to helping individuals with learning disabilities.

During a reading lesson, a first-grade teacher writes the word "boy" on the board. She points to the letter "b" and asks students to make the sound of the letter. Which of the following concepts is the teacher addressing? (A) Phonemic awareness (B) Pragmatics (C) Fluency (D) Letter-Sound correspondence

(D) Letter-Sound correspondence The understanding that a written symbol like the letter "b" makes a unique sound /b/ is the essence of letter-sound correspondence.

What does OHI stand for? (A) Only Healthy Individuals (B) Oppositional Hyperactivity Indicator (C) Organized Heart Implant (D) Other Health Impairment

(D) Other Health Impairment OHI stands for Other Health Impairment. This category includes a number of chronic or acute health problems that can adversely affect a child's educational performance. Competency 001

Researchers look at the strategies successful readers use to help them identify reading strategies to teach explicitly to struggling readers. Which of the following strategies is an example of a strategy most successful readers use during reading, often without consciously thinking about it? (A) Reading just to get to the end of the assignment (B) Skipping over words they don't recognize (C) Reading fiction and nonfiction the same way (D) Pausing every few paragraphs and mentally summarizing what just happened in the text

(D) Pausing every few paragraphs and mentally summarizing what just happened in the text Successful readers frequently check for understanding as they read a text. Often, they don't even realize they are doing so because it is such a quick mental review of what is happening and an evaluation of whether it makes sense in the context of the passage.

Under the Individuals with Disabilities Education Act, which of the following is NOT considered a disability category? (A) Speech/language impairment (B) Mental retardation (C) Specific learning disability (D) Post-traumatic stress disorder

(D) Post-traumatic stress disorder Under the Individuals with Disabilities Education Act, there are 13 disability categories.

A third-grade student with learning disabilities turns in the following piece of writing after 40 minutes of classroom writing time: After analyzing the writing, the student's Special Education coteacher notes strengths to encourage and weaknesses to target. Which of the following aspects of the student's writing would be considered a strength? (A) Syntax (B) Capitalization conventions (C) Writing fluency (D) Punctuation conventions

(D) Punctuation conventions One of the patterns evident in the writing sample is the writer's correct use of punctuation. The fact that every sentence is punctuated correctly is definitely a strength.

When formulating instructional objectives for a student's Individualized Education Program (IEP), what should the Annual Review and Dismissal (ARD) committee do? (A) Base the objectives exclusively on the grade-level TEKS (B) Construct the objectives based on what the student's parents want the child to be able to do (C) Let the student choose his or her own objectives (D) Review the student's current level of performance, and write reasonable objectives that reflect the grade-level TEKS as closely as possible and promote academic growth

(D) Review the student's current level of performance, and write reasonable objectives that reflect the grade-level TEKS as closely as possible and promote academic growth The ARD committee is responsible for writing and agreeing on reasonable objectives that will foster student growth and development while reflecting the grade-level TEKS as closely as possible.

A Special Education teacher arranges her classroom as the diagram shows below: What kind of activities is this classroom best set up for? (A) Whole group lectures (B) Sustained silent reading (C) Physical therapy (D) Small group and individual learning centers or stations

(D) Small group and individual learning centers or stations The diagram clearly shows a variety of tables and work stations. This allows the teacher to organize small-group and individual instruction while still maintaining order and classroom control. Success with learning centers and small-group instruction is often contingent on a welldesigned environment that allows students to work on projects and tasks without disturbing others.

For which students was the STAAR Alternate designed? (A) Students with learning disabilities (B) Students in alternative schools (C) Students with ADHD (D) Students with significant cognitive disabilities

(D) Students with significant cognitive disabilities The STAAR Alternate was designed for approximately 1% of Texas schoolchildren, specifically those students with such significant cognitive disabilities that they are not able to be tested fairly or accurately using a traditional multiple-choice format.

Which of the following students is most likely to benefit from an assistive technology device like a paper based computer pen? (A) Visual learner with physical disabilities (B) Auditory learner with social skills deficits (C) Nonverbal learner with behavioral challenges (D) Tactile learner with memory challenges

(D) Tactile learner with memory challenges A paper-based computer pen allows students to take notes and record the teacher's words at the same time. This is a great tool for tactile learners who learn through fine motor activities like writing but who also have memory issues. When the student uses the pen to review the notes, he or she can hear the recording of the classroom discussion simultaneously.

A Special Education teacher wants to share study strategies with students with learning disabilities and their families. He organizes an evening hands-on workshop where he presents information about different learning styles and models study strategies that fit each of the learning styles. What is this teacher trying to accomplish? (A) Earning a Teacher of the Year nomination (B) Showing the general education teachers that the Special Education department does more for students than the academic departments (C) Helping parents understand where their students have weaknesses (D) Teaching students and their families how to recognize their strengths and use those strengths to achieve success in the regular education environment, no matter who the teacher is

(D) Teaching students and their families how to recognize their strengths and use those strengths to achieve success in the regular education environment, no matter who the teacher is One of the most important things a Special Education teacher can do is teach students and their families strategies the student can use to succeed on his or her own. Knowing how one learns best allows a student to take control of his or her own learning. In addition, focusing on strengths is so important with students with disabilities who often find people and themselves focusing only on their weaknesses.

Which of the following behaviors would be a potential warning sign for an epileptic seizure? (A) Irritability (B) Extreme hunger (C) Nervous energy (D) Unexplained confusion, sleepiness, or weakness

(D) Unexplained confusion, sleepiness, or weakness Epileptic seizures have a number of potential warning signs. Unexplained confusion, sleepiness, and/or weakness are among those warning signs as are strange tastes or feelings, eyes rolling up, and drooling

A teacher in an autism unit has several students who become frustrated whenever their routines change. Two of them are nonverbal. What is one technique the teacher can use to help her students become more flexible? (A) Applied Behavior Analysis (B) Floortime (C) Circle time with calendar and weather (D) Visual schedule

(D) Visual schedule A visual schedule is a research-based technique that helps students with autism understand the plan for the day and what activities have to be completed before others can occur. This is especially helpful when a daily routine changes and a teacher wants to give forewarning. Reviewing the schedule and then removing the card representing each major activity after it is completed allows students to stay on task and have a sense of control over what is happening.

Research on effective teaching indicates that students who struggle with math respond positively to mathematics instruction that focuses on (A) improving basic math processing skills. (B) problem-solving. (C) conceptual understanding. (D) all of the above.

(D) all of the above. Evidence-based studies in mathematics instructional strategies have shown positive results from a group of methods that focus on improving math processing skills, problem solving, and conceptual understanding. The key to solid math instruction is to use all of these strategies, not limit instruction to any one area.

According to the Council for Exceptional Children's code of ethics, Special Educators and related professionals are bound to (A) maintaining challenging expectations for individuals with exceptionalities. (B) using evidence, instructional data, research, and professional knowledge to inform practice. (C) advocating for professional conditions and resources that will improve learning outcomes of individuals with exceptionalities. (D) all of the above.

(D) all of the above. Maintaining challenging expectations for individuals with exceptionalities; using evidence, instructional data, research, and professional knowledge to inform practice; and advocating for professional conditions and resources that will improve learning outcomes of individuals with exceptionalities are all part of CEC's code of ethics for Special Education professionals.

According to IDEA 2004 and CFR 300.43, transition services must be based on (A) a school's resources. (B) local community opportunities for students with disabilities. (C) parents' expectations for what their child can accomplish. (D) an individual student's needs, taking into account strengths, preferences, and interests.

(D) an individual student's needs, taking into account strengths, preferences, and interests. IDEA 2004 and its associated law, CFR 300.43, specifically state that transition services must be based on an individual student's needs and should take into account the student's strengths, preferences, and interests.

One advantage of implementing a research-based Response to Intervention (RTI) program from a Special Education teacher's perspective is that RTI (A) is the regular education teacher's responsibility, thus leaving the Special Education department with more time. (B) involves less work than Special Education diagnostic testing. (C) is easy to implement. (D) can potentially reduce overidentification of students with learning disabilities.

(D) can potentially reduce overidentification of students with learning disabilities. RTI provides increasingly intensive intervention as soon as learning deficits are demonstrated without the legal requirements for testing and eligibility that are part of Special Education. This early identification and intervention can correct learning deficits quickly and thus reduce the number of students who will later need Special Education services.

A fourth-grade student who recently immigrated to the United States with her family never looks her teacher in the eye. The teacher, after several months of documenting this behavior, refers the student for evaluation by the district's autism team. An important factor for the campus Special Education review team to consider is that (A) lack of eye contact is a sign of an autism spectrum disorder. (B) autism crosses cultural and linguistic borders. (C) it is overwhelming to be an immigrant and that such behaviors are to be expected. (D) cultural norms and expectations impact student behavior and teacher interpretation.

(D) cultural norms and expectations impact student behavior and teacher interpretation. In some cultures, it is disrespectful to look an authority figure in the eye. This may well be a factor in the scenario described. It is critical that Special Education teachers analyze how culturally and/or linguistically diverse backgrounds can impact behavior.

The reason that Section 504 of the Rehabilitation Act was ignored by state and local educational agencies for 20 years after it was passed in 1973 is that the law (A) didn't make sense to school boards. (B) was discriminatory. (C) was never signed by the President of the United States. (D) did not include provisions for federal funding or monitoring.

(D) did not include provisions for federal funding or monitoring. Section 504 of the Rehabilitation Act was ignored for 20 years by state and local educational agencies because it did not include provisions for federal funding or monitoring.

Marcus is a nine grader with intellectual disabilities. His transition needs evaluation indicates that he likes pets, especially dogs. He does not like doctors and becomes nervous around needles. Marcus is good at carrying out routine tasks. Based on this information, an appropriate supported employment opportunity for Marcus would be (A) taking animals at a veterinarian's office for walks. (B) dealing with customer issues at a customer service counter. (C) completing janitorial services at a local hospital. (D) feeding and washing dogs at an animal shelter.

(D) feeding and washing dogs at an animal shelter. Based on Marcus's interest in pets, particularly dogs, and his aptitude for routine tasks, a supported employment opportunity feeding and washing dogs at an animal shelter would be an appropriate supported employment opportunity.

A sixth-grade mathematics teacher uses her interactive whiteboard and individual student clickers to administer informal quizzes during her lessons at least twice a week. The most appropriate reason for this teacher to give students regular informal quizzes is to (A) get enough individual grades for an accurate average. (B) determine if a student should be recommended for Special Education testing. (C) utilize the expensive classroom technology. (D) identify skills that the teacher should reteach.

(D) identify skills that the teacher should reteach. Regular informal quizzes are examples of formative assessment. This assessment is best used to identify skills that students have not yet mastered and that the teacher, therefore, needs to reteach, preferably in a different way.

A sixth-grade social studies teacher is having difficulty motivating several students who are tactile/kinesthetic learners. The best way to deal with the problem is to (A) show a series of images related to the upcoming unit on the interactive whiteboard. (B) have the students read the next chapter in class, with each student expected to read a paragraph out loud. (C) play a recording of sounds associated with the content of the next chapter and have students guess what they are. (D) place sheets of chart paper around the room. On each one, write an open-ended question about a debatable issue related to the content of the next unit. Have students walk around the room and write their stance on each issue.

(D) place sheets of chart paper around the room. On each one, write an open-ended question about a debatable issue related to the content of the next unit. Have students walk around the room and write their stance on each issue. Tactile/kinesthetic learners retain information best when they are learning while moving. For example, a study strategy for a tactile/kinesthetic learner might be running laps while listening to recorded notes. Allowing students to move about the classroom while answering high-interest questions is a great way to motivate tactile/kinesthetic learners.

A Special Education teacher is administering a fluency assessment to a sixth grader. The teacher notes that the student reads 110 words per minute, which is below grade-level expectations. The area of fluency that the teacher is assessing is (A) accuracy. (B) expression. (C) syntax. (D) rate.

(D) rate. Rate is the speed at which a student reads, measured in words per minute. Typically, a sixth grade student should be reading 120-150 words per minute.

When talking to parents on the phone or at a parent-teacher conference, the most important thing a teacher can do is to (A) have documented examples for every problem the teacher has with the student. (B) have an answer for every question parents ask. (C) praise the parents for their outstanding parenting skills. (D) take time to listen to the parents' responses and concerns, and note anything that requires follow-up.

(D) take time to listen to the parents' responses and concerns, and note anything that requires follow-up. When talking with parents, the most important thing teachers can do is be respectful and responsive listeners. Parents have a wealth of information about their child and can often help problem-solve situations or anticipate where conflicts might occur. In any conversation, when participants feels they have an opportunity to speak and truly be listened to, they tend to feel more positive about the situation and more willing to compromise to find a solution.

Which of the following angles is an obtuse angle?

An obtuse angle is an angle that measures more than 90 degrees but less than 180 degrees. (A) is a 120-degree angle so it is obtuse.

A teacher presents students with the following pattern with the third and ninth shapes missing: Based upon this pattern, what would the ninth shape be?

The ninth shape would be a circle since we have established a repeating pattern of two circles, a square, and a triangle. The ninth shape would be the beginning of the third repetition, so it would start with a circle.

A student with an IEP is caught defacing a school wall. Based on the school's written code of conduct, the assistant principal suspends the student from school for three days. The student has not been suspended previously that school year, but the parents protest the suspension, saying that the student's status as a Special Education student prevents him from being suspended. The most appropriate response is that (A) the school district should override the suspension since the student receives Special Education services. (B) the principal should ask the assistant principal to tell the parents that if they pay for the damages, the student will not be suspended. (C) the suspension should be changed to in-school suspension so that the parents do not have to arrange for child care. (D) the assistant principal should call a meeting with the parents with another administrator present and point out that disciplinary sanctions of less than 10 days are not considered a change of placement under IDEA and are, therefore, legal.

the assistant principal should call a meeting with the parents with another administrator present and point out that disciplinary sanctions of less than 10 days are not considered a change of placement under IDEA and are, therefore, legal. Under IDEA, disciplinary sanctions are not considered a change in placement and are not subject to this restriction provided the student has not been suspended for more than 10 days cumulatively during the school year. To help ensure a good relationship between the school and parents, the assistant principal should share this information with the parents in a nonconfrontational context with a witness


Kaugnay na mga set ng pag-aaral

MGMT 340 - Exam 2 Practice Test Ch. 11

View Set

SCMT 489: Purchasing and Supply Management Exam 2 Chapter 11,12,13

View Set

Chapter 38: Nursing Care of the Child With an Alteration in Intracranial Regulation/Neurologic Disorder

View Set

PATH 370 - W7 Check Your Understanding

View Set